Anda di halaman 1dari 4

Hungerford: Algebra II.5.

Sylow Theorems

1. If N G and N, G/N are both p-groups, then G is a p-group. Proof: Arbitrarily pick a G. Then a = aN G/N . Since G/N is a p-group, |a| = pn , for some positive integer n. Thus (aN )p = N , and so ap N . Since N is a p-group, for some integer m > 0, (ap )p = e. This means that ap and so G is a p-group. 2. If |G| = pk , H G and |H| > 1, then |H C(G)| > 1. Proof Let G act on H by conjugation. Then the xed points HG = {h H | ghg 1 = h, g G} = H C(G). Since G is a p-group, |HG | |H| 0 (mod p). But |HG | 1. 3A. If H C(G), then H G. Proof We verify this by denition. x G, and h H, since h H C(G), xhx1 = hxx1 = h H. 3. Let |G| = pn , where p is a prime and n > 0 is an integer. For each k, 0 k n, G has a normal subgroup of order pk . Proof We rst prove the following. Claim 1 G has a normal subgroup of order p. In fact, since G is a p-group, its center C(G) = {e}. Therefore, C(G) is a p-subgroup of G. By Cauchys lemma, there is an element a C(G) such that |a| = p. Then N = a is a a subgroup of order p. Since every subgroup of C(G) is normal in G, N G. This proves the claim. Next, we shall show that for each k, 0 k n, G has a normal subgroup of order pk , by induction on k. When k = 0, {e} is a normal subgroup of order 1. Assume that G has a normal subgroup K with |K| = pk , and 1 k < n. Consider G = G/K. By Lagrange, |G | = pnk , and so G is a p-group. By Claim 1, G has a normal subgroup N = aH projection, and let H = 1 (H ) = {g G : gK N }. 1 such that |N | = p. Let : G G be the canonical
n m n+m n n

= e,

Then by Theorem I.5.11, and since H G , H G. Moreover, H = H/K, and so by Lagrange, |H| = |K|p = pk+1 . Therefore, the statement is proved by induction. 4. If G is an innite p-group (a group in which every non identity element has order a power of a prime p), then either G has a subgroup of order pn for each integer n > 0, or there exists an integer m > 0 such that every nite subgroup of G has order at most pm . Proof Suppose that no such an m exists. Then for each integer n > 0, there is an integer N > n such that G has a subgroup H of order pN . But then, by Sylow, H has a subgroup of order pn . 5. If P is a normal Sylow-p-subgroup of a nite group G and f : G G is an endomorphism, then f (P ) < P . Proof: Since P G, and since the homomorphism image of a group is also a group, then f (P ) G. Since P is a p-group, x P , |x| = pn is a power of p. Since f is a homomorphism, |f (x)| is a factor of |x|, and so it is also a power of p. It follows that f (P ) is also a p-subgroup of G, and so for some Q Sylp (G) such that f (P ) Q. By the 2nd Sylow Theorem, and since P G, Sylp (G) = {P }, and so we must have Q = P. 6. If H is a normal subgroup of order pk of a nite group G, then H is contained in every Sylow p-subgroup of G. Proof Let P be a Sylow p-subgroup of G. By Sylow, H, as a p-subgroup, is contained in a Sylow p-subgroup Q of G. By Sylow, P = xQx1 for some x G. Thus H = xHx1 xQx1 = P . 9. If |G| = pn q with p > q primes, then G contains a unique normal subgroup of index q. Proof By Sylow, G has a Sylow p-subgroup P with [G : P ] = q. Since p > q, q 1 (mon p), and so by Sylow, P is the only one Sylow p-subgroup of G. Hence P G.

10. Every group of order 12, 28, 56 and 200 must contain a normal Sylow subgroup. Proof: We shall use the fact that if G is not isomorphic to Zp , and if G has only one Sylow p-subgroup, then the only Sylow p-subgroup os a normal Sylow subgroup of G.

(i) |G| = 12 = (2)2 (3). Let n3 = |Syl3 (G)|. By 3rd Sylow Theorem, n3 {1, 4}. If n3 = 1, then done. We assume that n3 = 4, then let Syl3 (G) = {H1 , H2 , H3 , H4 }. Then there are 4(3 1) = 8 elements of order 3 in G, and so G must have exactly one Sylow 2-subgroup. (ii) |G| = 28 = (2)2 (7). Let n7 = |Syl7 (G)|. By 3rd Sylow Theorem, n7 is a factor of 22 , and n7 1 (mod 7). Thus n7 = 1. (iii) |G| = 56 = (2)3 (7). Let n7 = |Syl7 (G)|. By 3rd Sylow Theorem, n7 {1, 8}. If n7 = 1, then done. We assume that n7 = 8, then let Syl7 (G) = {H1 , H2 , , H8 }. Then there are 8(7 1) = 48 elements of order 7 in G, and so G must have exactly one Sylow 2-subgroup. (iv) |G| = 200 = (2)3 (5)2 . Let n5 = |Syl5 (G)|. By 3rd Sylow Theorem, n5 is a factor of 23 , and n5 1 (mod 1). Thus n5 = 1. 11. How many elements of order 7 are there in a simple group of order 168? Solution |G| = 168 = (2)2 (3)(7). Let n7 = |Syl7 (G)|. By 3rd Sylow Theorem, n7 must divide (2)2 (3) and n7 1 (mod 7), and so n7 {1, 8}. Since G is simple, n7 = 1. Thus n7 = 8, and we may assume that Syl7 G = {H1 , H2 , , H8 }. Since for each i, |Hi | = 7, and Hi Z7 . If follows that if i = j, then Hi Hj = {e}. Hence the number of order 7 = elements is 8(7 1) = 48. 13. Every group G of order p2 (p prime) is abelian. Proof Since G is a p-group, the center C(G) = {e}. By Lagrange, either |C(G)| = p2 , whence G is abelian; or |C(G)| = p. If |V (G)| = p, then G/C(G) is cyclic, and so G is abelian (by Exercise II-4.9). Therefore, we must have C(G) = G, a contradiction.

Supplemental Problems

1. Let H G with [G : H] < . Show that H has a nite number of conjugates in G. Proof Let S denote the set of all subgroups of G and let G acts on S by conjugation: g G and K S, g(K) = gKg 1 . Then the orbit of the action OH is OH = {gHg 1 | g G}. Consider the subgroup in G that xes H: GH = {g G | g(H) = H} = {g G | gHg 1 = H}. Then H GH G, and so [G : GH ] [G : H] < . One can check that the map (gHg 1 ) = gGH is a well dened bijection between OH and G/GH , and so |OH | = [G : GH ] [G : H] < . 2. No group of order 36 is simple. Proof Suppose that n3 = |Syl3 (G)| = 4, (otherwise n3 = 1 and G is not simple). Let Syl3 (G) = {H1 , H2 , H3 , H4 } and H = H1 H2 such that H is maximal among all the intersections of two members in Syl3 (G). Then 48 = |G| |H1 H2 | = |H1 | |H2 | 81 = , |H| |H|

implying |H| = 3. Let N = NG (H). Then by the maximality of H, for every Hi Syl3 (G) such that N Hi , we have Hi N Syl3 (N ) and Hi N properly contains H (see Theorem (3.2) in Lecture Notes: Theorems on Sylow p-subgroups). It follows that both H1 , H2 Syl3 (N ), and so 36 |N | |H1 H2 | = |H1 | |H2 | 81 = = 27. |H| 3

But as N G, Lagrange said that |N | must be a factor of 36, and so N = G.

Anda mungkin juga menyukai